Đến nội dung

Hình ảnh

Đề thi chọn đội tuyển Olympic quốc tế (TST) năm 2023


  • Please log in to reply
Chủ đề này có 6 trả lời

#1
nhungvienkimcuong

nhungvienkimcuong

    Thiếu úy

  • Hiệp sỹ
  • 669 Bài viết

Đề thi chọn đội tuyển Olympic quốc tế năm 2023

Thời gian: 270 phút

 

Ngày thi thứ nhất: 13/04/2023

 

Bài 1. Cho hai lớp học, lớp $A$ có $m$ học sinh và lớp $B$ có $n$ học sinh $(m,\ n>1)$. Học sinh của hai lớp ngồi quanh một bàn tròn và mỗi em học sinh X được cô giáo phát kẹo bằng với số bạn ngồi liên tiếp kề bên trái X và cùng lớp với X (nếu X không có những bạn như vậy thì X không có kẹo). Những người có cùng số kẹo được cô giáo phân chia vào cùng một nhóm.

a) Hỏi số người đông nhất của một nhóm có thể là bao nhiêu?

b) Nếu không xét nhóm mà học sinh không có kẹo thì số người đông nhất của một nhóm có thể là bao nhiêu?

 

Bài 2. Xét các hàm số sau đây trên tập số thực khác $0$:

\[P(x)=\left(x^2-1\right)^{2023},\quad Q(x)=(2x+1)^{14},\quad R(x)=\left(2x+1+\frac{2}{x}\right)^{34}.\]

Giả sử ban đầu có một danh sách gồm đúng hai hàm trong các hàm đã cho. Mỗi thao tác được phép cộng, trừ, nhân các hàm trong danh sách đó lại với nhau (hoặc lấy lũy thừa với số mũ nguyên dương một hàm trong đó). Ta cũng có thể cộng, trừ, nhân một hàm với một số thực tùy ý để tạo ra hàm mới và đưa vào danh sách. Quá trình trên có thể thực hiện nhiều lần. Chứng minh rằng từ danh sách ban đầu là hai hàm bất kì trong ba hàm đã cho, ta không thể thu được hàm còn lại.

 

Bài 3. Cho tam giác $ABC$ nhọn không cân nội tiếp đường tròn $(O)$. Các đường cao $BE,\ CF$ của tam giác $ABC$ cắt nhau ở trực tâm $H$ và $M$ là trung điểm $AH$. Gọi $K$ là hình chiếu của $H$ lên $EF$. Đường thẳng không đi qua $A$ và song song với $BC$ cắt cung nhỏ $AB,\ AC$ lần lượt tại các điểm $P,\ Q$. Chứng minh rằng tiếp tuyến tại $E$ của đường tròn ngoại tiếp tam giác $CQE$ và tiếp tuyến tại $F$ của đường tròn ngoại tiếp tam giác $BPF$ cắt nhau trên đường thẳng $MK$.

 

 

Ngày thi thứ hai: 14/04/2023

 

Bài 4. Cho hai số nguyên dương $a,\ b$ nguyên tố cùng nhau với $b$ lẻ và $a>2$. Xét dãy số $(x_n)$ xác định bởi $x_0=2,\ x_1=a$ và $x_{n+2}=ax_{n+1}+bx_n$ với mọi $n$ nguyên dương. Chứng minh rằng

a) Nếu $a$ chẵn thì không tồn tại các số nguyên dương $m,\ n,\ p$ để $\frac{x_m}{x_nx_p}$ là số nguyên.

b) Nếu $a$ lẻ thì không tồn tại các số nguyên dương $m,\ n,\ p$ sao cho $mnp$ chẵn và $\frac{x_m}{x_nx_p}$ là số chính phương.

 

Bài 5. Cho tứ giác lồi $ABCD$ có $\widehat{B}<\widehat{A}<90^{\circ}$. Gọi $I$ là trung điểm của $AB$ và $S$ là giao điểm của $AD$ với $BC$. Xét $R$ là một điểm thay đổi nằm bên trong tam giác $SAB$ sao cho $\widehat{ASR}=\widehat{BSR}$. Trên các đường thẳng $AR,\ BR$ lần lượt lấy các điểm $E,\ F$ sao cho $BE$ và $AF$ cùng song song với $RS$. Giả sử $EF$ cắt đường tròn ngoại tiếp tam giác $SAB$ tại các điểm $H,\ K$. Trên đoạn $AB$, lấy các điểm $M$ và $N$ sao cho $\widehat{AHM}=\widehat{BHI}$ và $\widehat{BKN}=\widehat{AKI}$.

a) Chứng minh rằng tâm $J$ của đường tròn ngoại tiếp tam giác $SMN$ thuộc một đường thẳng cố định.

b) Trên $BE,\ AF$ lần lượt lấy các điểm $P,\ Q$ sao cho $CP$ song song với $SE$ và $DQ$ song song với $SF$. Các đường thẳng $SE,\ SF$ cắt lại đường tròn $(O)$ theo thứ tự tại $U,\ V$. Gọi $G$ là giao điểm của $AU$ với $BV$. Chứng minh rằng đường trung tuyến đỉnh $G$ của tam giác $GPQ$ luôn đi qua một điểm cố định.

 

Bài 6. Cho số nguyên $n\ge 3$ và tập hợp $A=\{1,2,\dots,n\}$. Xác định số $k$ lớn nhất sao cho với mỗi bộ $k$ tập con có $3$ phần tử của $A$, luôn tô màu được mỗi phần tử của $A$ bởi một trong hai màu xanh hoặc đỏ (mỗi phần tử một màu) để không có tập con nào trong $k$ tập con trên có ba phần tử cùng màu.

 

 

Nguồn: Hướng tới Olympic Toán VN (nhóm facebook)


Bài viết đã được chỉnh sửa nội dung bởi nhungvienkimcuong: 15-04-2023 - 13:53

Đừng khóc vì chuyện đã kết thúc hãy cười vì chuyện đã xảy ra ~O) 
Thật kì lạ anh không thể nhớ đến tên mình mà chỉ nhớ đến tên em :wub:
Chúa tạo ra vũ trụ của con người còn em tạo ra vũ trụ của anh :ukliam2:


#2
hxthanh

hxthanh

    Tín đồ $\sum$

  • Hiệp sỹ
  • 3916 Bài viết



Bài 6. Cho số nguyên $n\ge 3$ và tập hợp $A=\{1,2,\dots,n\}$. Xác định số $k$ lớn nhất sao cho với mỗi bộ $k$ tập con có $3$ phần tử của $A$, luôn tô màu được mỗi phần tử của $A$ bởi một trong hai màu xanh hoặc đỏ (mỗi phần tử một màu) để không có tập con nào trong $k$ tập con trên có ba phần tử cùng màu.

Để tăng tương tác mình xin mạnh dạn “dự đoán”
TH1: $n=3$ hiển nhiên $\max k_3=1$
TH2: $n=4$ có tất cả ${4\choose 3}=4$ tập con của $A$. Ta có thể tô màu đỏ 2 phần tử của $A$ và 2 phần tử còn lại màu xanh. Suy ra $\max k_4=4$
Tiếp theo ta “thử” dự đoán $\max k_n=\frac{n-2}{2}\left\lfloor\frac{n^2}{4}\right\rfloor$
Bây giờ thử xem ta giải quyết vấn đề với $n=5$ như thế nào!
Giả sử ta chia tập $A$ thành 2 tập $A_1=\{1,2,3\}$ và $A_2=\{4,5\}$. Tô $A_1$ màu đỏ còn $A_2$ màu xanh.
Khi đó ta có thể chọn ra các tập con $3$ phần tử thoả mãn bằng cách chọn tối đa 2 phần tử trong mỗi tập $A_i$.
Số tập con chọn được bằng $3.2+3.1=9$
Dễ dàng chứng minh đây là cách chọn tối đa khi phân chia tập $A$
Công việc tiếp theo ta sẽ chứng minh

Liệu có đúng không? Mời các bạn comment…

Bài viết đã được chỉnh sửa nội dung bởi hxthanh: 15-04-2023 - 11:41


#3
hxthanh

hxthanh

    Tín đồ $\sum$

  • Hiệp sỹ
  • 3916 Bài viết
Chợt nhận ra suy nghĩ của mình sai bét!
$k_n$ không thể “tuyến tính” theo $n$
Vì nếu $n\ge 6$. Nếu chọn ra đủ ${5\choose 3}=10$ tập con 3 phần tử của 5 số bất kỳ thì hết cách tô màu!
Tạm thời kết luận $k_n\le 9$

#4
chuyenndu

chuyenndu

    Trung sĩ

  • Thành viên
  • 177 Bài viết

bài 6

ở trên fb thì em thấy với n>6 thì k=6



#5
Hoang72

Hoang72

    Thiếu úy

  • Điều hành viên OLYMPIC
  • 539 Bài viết

Bài 4:

Dễ thấy công thức tổng quát của $(x_n)$ là: $x_n = \left(\frac{\sqrt{a^2+4b} +a}{2}\right)^n + \left(\frac{-\sqrt{a^2+4b} +a}{2}\right)^n$

Trước tiên, ta có một số nhận xét sau: 

Nhận xét 1: $(x_n, b) = 1,\forall n\in\mathbb N^*$.

Chứng minh: Từ giả thiết ta có ngay $(x_0,b)=(x_1,b) = 1$. 

Mặt khác $x_{n+2} \equiv ax_{n+1}\pmod b,\forall n\in\mathbb N$ nên bằng quy nạp, dễ dàng suy ra đpcm.

Nhận xét 2: $\forall m,n\in\mathbb N^*: x_n\mid x_m\Rightarrow \begin{cases} n\mid m \\ v_2(n) = v_2(m)\end{cases}$.

Chứng minh: Đặt $t$ là số dư của $m$ khi chia cho $2n$.

Nhận thấy: $x_m = x_nx_{m-n} - (-b)^n\left(\left(\frac{\sqrt{a^2+4b} +a}{2}\right)^{m-2n} + \left(\frac{-\sqrt{a^2+4b} +a}{2}\right)^{m-2n}\right)$.

Do đó nếu $m\geq 2n$ thì $x_n\mid (-b)^nx_{m-2n} \Rightarrow x_n\mid x_{m-2n}$.

Cứ tiếp tục như thế, ta có: $x_n\mid x_t\Rightarrow n\leq t$.

Mặt khác, ta có: $x_t = x_nx_{t-n} - (-b)^{t-n}x_{2n - t}\Rightarrow x_n\mid x_{2n-t} \Rightarrow n\leq 2n - t\Rightarrow n \geq t$.

Như vậy, $n = t$. Suy ra $2n\mid m - n$, hay $\begin{cases} n\mid m \\ v_2(n) = v_2(m)\end{cases}$.

Trở lại bài toán:

a) Vì $2\mid a$, $2\nmid b$ nên $x_{n+2}\equiv bx_n\equiv x_n\pmod 2,\forall n\in\mathbb N$

$\Rightarrow 2\mid x_n,\forall n\in\mathbb N$.

Từ đó, $4\mid ax_n$ nên $x_{n+2}\equiv bx_n\pmod 4,\forall n\in\mathbb N$. $(*)$

Từ $(*)$ dễ có $v_2(x_n) = 1,\forall n\in\mathbb N, 2\mid n$.

Đồng thời, bằng quy nạp, dễ dàng chứng minh được $v_2(x_n) = v_2(a),\forall n\in\mathbb N, 2\nmid n$.

Giả sử tồn tại $m,n,p\in\mathbb N^*$ mà $\frac{x_m}{x_nx_p}\in\mathbb Z$.

Nếu $n$ hoặc $p$ lẻ thì $v_2(x_nx_p)\geq v_2(a) + 1\geq v_2(x_m)$, vô lí,

Do đó $n,p$ đều chẵn. Hay $4\mid x_m\Rightarrow 2\nmid m$.

Mặt khác, $x_n\Rightarrow x_m\Rightarrow n\mid m$ (Từ Nhận xét 2).

Điều này rõ ràng vô lí. Ta có đpcm.

b) Giả sử tồn tại $m,n,p\in\mathbb N^*$ thoả mãn $\begin{cases} 2\mid mnp \\ \dfrac{x_m}{x_nx_p}\text{ là số chính phương}\end{cases}$.

Thế thì, $\begin{cases} x_n\mid x_m \\ x_p\mid x_m\end{cases}\Rightarrow v_2(m)=v_2(n)=v_2(p)$

$\Rightarrow m,n,p$ chẵn.

Nhận thấy với mọi $k\in\mathbb Z$ thì: $x_{2k} = x_k^2 - 2(-b)^k$.

Vì $b$ lẻ nên suy ra $x_{2k}\equiv 2,3\pmod 4$.

Suy ra $x_m,x_n,x_p\equiv 2,3\pmod 4$.

TH1: $x_m\equiv 3\pmod 4$: Khi đó $2\nmid x_n,x_p\Rightarrow x_n,x_p\equiv 3\pmod 4$

$\Rightarrow \frac{x_m}{x_nx_p}\equiv 3\pmod 4$, vô lí.

TH2: $x_m\equiv 2\pmod 4$: Đặt $y_k = x_{2k},\forall k\in\mathbb N$.

Khi đó: $\begin{cases} y_0 = 2 \\ y_1 = a^2 + 2b \\ y_{n+2} = (a^2 + 2b)y_{n+1} - b^2y_n,\forall n\in\mathbb N\end{cases}$.

Vì $a,b$ lẻ nên $a^2+2b\equiv 3\pmod 4$. Suy ra dãy tuần hoàn mod $4$ của $(y_n)$ là: $2,3,3,2,3,3,...$.

$\bullet$ $a^2+2b\equiv 3\pmod 8$: Khi đó dãy tuần hoàn mod $8$ của $(y_n)$ là: $2,3,7,2,7,3,2,3,7,...$.

$\bullet$ $a^2+2b\equiv 7\pmod 8$: Khi đó dãy tuần hoàn mod $8$ của $(y_n)$ là: $2,7,7,2,7,7,...$.

Do đó $y_n\equiv 2\pmod 4\Leftrightarrow y_n\equiv 2\pmod 8$.

Điều này dẫn đến $x_m\equiv 2\pmod 8$.

Và $x_n,x_p$ hoặc chia $4$ dư $3$, hoặc chia $8$ dư $2$.

Xét các trường hợp ta thấy $\frac{x_m}{x_nx_p}\equiv 2,3\pmod 4$, vô lí.

Ta có điều phải chứng minh.


Bài viết đã được chỉnh sửa nội dung bởi Hoang72: 17-04-2023 - 08:33


#6
chuyenndu

chuyenndu

    Trung sĩ

  • Thành viên
  • 177 Bài viết

bài 2

P,Q không sinh ra R là đương nhiên

 

P', R' có nghiệm {1,-1} nhưng Q' thì không. nên P,R không sinh ra Q

 

chọn số phức z: $(2z+1)^2,z+\frac{1}{z}$ là số thực và $z^2-1$ không phải số thực thì suy ra Q,R không sinh ra P

mò thấy $z=\frac{-1}{2}+\frac{\sqrt{3}}{2}i$ ok


Bài viết đã được chỉnh sửa nội dung bởi chuyenndu: 19-04-2023 - 05:49


#7
perfectstrong

perfectstrong

    $LOVE(x)|_{x =\alpha}^\Omega=+\infty$

  • Quản lý Toán Ứng dụng
  • 4991 Bài viết

Bài 6. Cho số nguyên $n\ge 3$ và tập hợp $A=\{1,2,\dots,n\}$. Xác định số $k$ lớn nhất sao cho với mỗi bộ $k$ tập con có $3$ phần tử của $A$, luôn tô màu được mỗi phần tử của $A$ bởi một trong hai màu xanh hoặc đỏ (mỗi phần tử một màu) để không có tập con nào trong $k$ tập con trên có ba phần tử cùng màu.

Em giải thử như sau:

Bài toán
Cho một đồ thị $G$ hoàn chỉnh có các đỉnh được đánh số thứ tự $1, 2,\ldots,n$. Tìm $k$ lớn nhất để với mọi $k$ tam giác chọn từ $G$, luôn tồn tại một cách tô màu $G$ (xanh/đỏ trên mỗi đỉnh) để không có tam giác nào được chọn lại có 3 đỉnh cùng màu.

- $n=3 \Rightarrow k =1$.

- $n=4 \Rightarrow k = 4$: tô $1,3$ đỏ và $2, 4$ xanh.

- $n=5 \Rightarrow k = 9$: vì $k \le C_{5}^{3} =  10$, mà nếu $k=10$, tức là chọn hết mọi thì vô lý vì theo Dirichlet, luôn tồn tại 3 đỉnh cùng màu. $k=9$ thì chỉ cần tô cùng màu 3 đỉnh của tam giác không được chọn là thỏa.

- $n \ge 6$. Ta chứng minh $k < 10$. Thật vậy, với mọi $k \ge 10$, ta có thể chọn một đồ thị con $K_5$ từ $G$. Mà $K_5$ theo chứng minh ở trên thì luôn có một tam giác đơn sắc.

Do đó, $k \le 9$. Bây giờ chỉ cần xây dựng cách tô để thỏa mãn $k=9$, hoặc là phủ nhận :D


Bài viết đã được chỉnh sửa nội dung bởi perfectstrong: 19-04-2023 - 19:36

Luôn yêu để sống, luôn sống để học toán, luôn học toán để yêu!!! :D
$$\text{LOVE}\left( x \right)|_{x = \alpha}^\Omega = + \infty $$
I'm still there everywhere.




3 người đang xem chủ đề

1 thành viên, 2 khách, 0 thành viên ẩn danh